What is the problem here (all integers are irrational proof…I think so)?How do we prove $n^n mid m^m Rightarrow n mid m$?Catalan constant is irrational. What is wrong with this proof?The contradiction method used to prove that the square root of a prime is irrationalComplex Exponential False “Proof” That All Integers Are $0$Understanding the proof of “$sqrt2$ is irrational” by contradiction.Prime Factors of the Composit Terms of Arithmetic ProgressionsIs there a quicker proof to show that $2^10^k equiv 7 pmod9$ for all positive integers $k$?Prove the sqrt of 4 is irrational, where did I go wrong?A Proof of the Fundamental Theorem of ArithmeticInteger factors of rational and irrational numbersProof verification: Prove $sqrtn$ is irrational.

Why doesn't philosophy have higher standards for its arguments?

How can I leave a car for someone if we can't meet in person?

Adjusting vertical spacing in fractions?

Is the Malay "garam" (salt) related to the Latin "garum" (fish sauce)?

Is this artwork (used in a video game) real?

Is it ethical for a company to ask its employees to move furniture on a weekend?

Is it OK to use personal email ID for faculty job applications or should we use (current) institute's ID

What powers the air required for pneumatic brakes in aircraft?

How do I query for system views in a SQL Server database?

Cine footage fron Saturn V launch's

The Binomial Elks Club

Alphanumeric Line and Curve Counting

What happens if there is no space for entry stamp in the passport for US visa?

How could an animal "smell" carbon monoxide?

Cauchy reals and Dedekind reals satisfy "the same mathematical theorems"

Can a Resident Assistant Be Told to Ignore a Lawful Order?

Why should I cook the flour first when making bechamel sauce?

Why does "git status" show I'm on the master branch and "git branch" does not?

Why don't commercial aircraft adopt a slightly more seaplane-like design to allow safer ditching in case of emergency?

Is straight-up writing someone's opinions telling?

What is the meaning of [[:space:]] in bash?

Why does FFmpeg choose 10+20+20 ms instead of an even 16 ms for 60 fps GIF images?

How could a medieval fortress manage large groups of migrants and travelers?

Sending a photo of my bank account card to the future employer



What is the problem here (all integers are irrational proof…I think so)?


How do we prove $n^n mid m^m Rightarrow n mid m$?Catalan constant is irrational. What is wrong with this proof?The contradiction method used to prove that the square root of a prime is irrationalComplex Exponential False “Proof” That All Integers Are $0$Understanding the proof of “$sqrt2$ is irrational” by contradiction.Prime Factors of the Composit Terms of Arithmetic ProgressionsIs there a quicker proof to show that $2^10^k equiv 7 pmod9$ for all positive integers $k$?Prove the sqrt of 4 is irrational, where did I go wrong?A Proof of the Fundamental Theorem of ArithmeticInteger factors of rational and irrational numbersProof verification: Prove $sqrtn$ is irrational.






.everyoneloves__top-leaderboard:empty,.everyoneloves__mid-leaderboard:empty,.everyoneloves__bot-mid-leaderboard:empty margin-bottom:0;








7












$begingroup$


Let us assume $a$ is an integer which is rational which implies $a=p/q$ (where $p$ and $q$ are integers and $q$ not equal to $0$). If $p$ and $q$ are not coprime, let us simplify the fraction so this it is (I don't know how to talk like mathematicians).



Which implies, $$a=b/c$$ (where $b$ and $c$ are coprime integers). Squaring on both sides,
beginalign
a^2&=b^2/c^2\
a^2c^2&=b^2
endalign

So $a^2$ is a factor of $b^2$, and also of $b$, due to the uniqueness of the fundamental theorem of arithmetic.
So,
beginalign
b &=a^2d tagwhere $d$ is an integer\
b^2 &= a^4d^2
endalign

But $b^2=a^2c^2$ So,
beginalign
a^2c^2 &= a^4d^2\
c^2 &= a^2d^2
endalign



So, $a^2$ is a factor of $c^2$ and $c$ due to the fundamental theorem of arithmetic. So $b$ and $c$ have $a^2$ as a common factor. But this contradicts the fact that $b$ and $c$ are coprime. This is because we have taken $a$ as a rational integer, so $a$ cannot be a rational integer.



What's wrong here (genuinely asking)?










share|cite|improve this question











$endgroup$







  • 11




    $begingroup$
    "a^2 is a factor b^2 and b" You are right $a^2$ divides $b^2$, but why would it divide $b$?
    $endgroup$
    – Wojowu
    Jul 7 at 16:06






  • 7




    $begingroup$
    First error I spotted is that $a^2$ need not be a factor of $b$ just because it is a factor of $b^2$. Indeed, if $a=2$ and $b=2$, then $a^2=4$ is a factor of $b^2=4$ but $a^2=4$ is not a factor of $b=2$.
    $endgroup$
    – Dave
    Jul 7 at 16:06






  • 3




    $begingroup$
    Also, it is clear that integers are rational because given any integer $a$ we can write $a=fraca1$.
    $endgroup$
    – Dave
    Jul 7 at 16:08






  • 4




    $begingroup$
    If b and c are coprime, and a=b/c, then what does c have to be?
    $endgroup$
    – pokep
    Jul 7 at 17:33






  • 3




    $begingroup$
    @Toolazytothinkofaname It is rather simple to spot the error. Substitute the variables by actual integers and find where the proof is wrong.
    $endgroup$
    – miracle173
    Jul 8 at 14:18

















7












$begingroup$


Let us assume $a$ is an integer which is rational which implies $a=p/q$ (where $p$ and $q$ are integers and $q$ not equal to $0$). If $p$ and $q$ are not coprime, let us simplify the fraction so this it is (I don't know how to talk like mathematicians).



Which implies, $$a=b/c$$ (where $b$ and $c$ are coprime integers). Squaring on both sides,
beginalign
a^2&=b^2/c^2\
a^2c^2&=b^2
endalign

So $a^2$ is a factor of $b^2$, and also of $b$, due to the uniqueness of the fundamental theorem of arithmetic.
So,
beginalign
b &=a^2d tagwhere $d$ is an integer\
b^2 &= a^4d^2
endalign

But $b^2=a^2c^2$ So,
beginalign
a^2c^2 &= a^4d^2\
c^2 &= a^2d^2
endalign



So, $a^2$ is a factor of $c^2$ and $c$ due to the fundamental theorem of arithmetic. So $b$ and $c$ have $a^2$ as a common factor. But this contradicts the fact that $b$ and $c$ are coprime. This is because we have taken $a$ as a rational integer, so $a$ cannot be a rational integer.



What's wrong here (genuinely asking)?










share|cite|improve this question











$endgroup$







  • 11




    $begingroup$
    "a^2 is a factor b^2 and b" You are right $a^2$ divides $b^2$, but why would it divide $b$?
    $endgroup$
    – Wojowu
    Jul 7 at 16:06






  • 7




    $begingroup$
    First error I spotted is that $a^2$ need not be a factor of $b$ just because it is a factor of $b^2$. Indeed, if $a=2$ and $b=2$, then $a^2=4$ is a factor of $b^2=4$ but $a^2=4$ is not a factor of $b=2$.
    $endgroup$
    – Dave
    Jul 7 at 16:06






  • 3




    $begingroup$
    Also, it is clear that integers are rational because given any integer $a$ we can write $a=fraca1$.
    $endgroup$
    – Dave
    Jul 7 at 16:08






  • 4




    $begingroup$
    If b and c are coprime, and a=b/c, then what does c have to be?
    $endgroup$
    – pokep
    Jul 7 at 17:33






  • 3




    $begingroup$
    @Toolazytothinkofaname It is rather simple to spot the error. Substitute the variables by actual integers and find where the proof is wrong.
    $endgroup$
    – miracle173
    Jul 8 at 14:18













7












7








7





$begingroup$


Let us assume $a$ is an integer which is rational which implies $a=p/q$ (where $p$ and $q$ are integers and $q$ not equal to $0$). If $p$ and $q$ are not coprime, let us simplify the fraction so this it is (I don't know how to talk like mathematicians).



Which implies, $$a=b/c$$ (where $b$ and $c$ are coprime integers). Squaring on both sides,
beginalign
a^2&=b^2/c^2\
a^2c^2&=b^2
endalign

So $a^2$ is a factor of $b^2$, and also of $b$, due to the uniqueness of the fundamental theorem of arithmetic.
So,
beginalign
b &=a^2d tagwhere $d$ is an integer\
b^2 &= a^4d^2
endalign

But $b^2=a^2c^2$ So,
beginalign
a^2c^2 &= a^4d^2\
c^2 &= a^2d^2
endalign



So, $a^2$ is a factor of $c^2$ and $c$ due to the fundamental theorem of arithmetic. So $b$ and $c$ have $a^2$ as a common factor. But this contradicts the fact that $b$ and $c$ are coprime. This is because we have taken $a$ as a rational integer, so $a$ cannot be a rational integer.



What's wrong here (genuinely asking)?










share|cite|improve this question











$endgroup$




Let us assume $a$ is an integer which is rational which implies $a=p/q$ (where $p$ and $q$ are integers and $q$ not equal to $0$). If $p$ and $q$ are not coprime, let us simplify the fraction so this it is (I don't know how to talk like mathematicians).



Which implies, $$a=b/c$$ (where $b$ and $c$ are coprime integers). Squaring on both sides,
beginalign
a^2&=b^2/c^2\
a^2c^2&=b^2
endalign

So $a^2$ is a factor of $b^2$, and also of $b$, due to the uniqueness of the fundamental theorem of arithmetic.
So,
beginalign
b &=a^2d tagwhere $d$ is an integer\
b^2 &= a^4d^2
endalign

But $b^2=a^2c^2$ So,
beginalign
a^2c^2 &= a^4d^2\
c^2 &= a^2d^2
endalign



So, $a^2$ is a factor of $c^2$ and $c$ due to the fundamental theorem of arithmetic. So $b$ and $c$ have $a^2$ as a common factor. But this contradicts the fact that $b$ and $c$ are coprime. This is because we have taken $a$ as a rational integer, so $a$ cannot be a rational integer.



What's wrong here (genuinely asking)?







elementary-number-theory fake-proofs






share|cite|improve this question















share|cite|improve this question













share|cite|improve this question




share|cite|improve this question








edited Jul 8 at 14:00









Peter Mortensen

5783 silver badges10 bronze badges




5783 silver badges10 bronze badges










asked Jul 7 at 16:02









ToolazytothinkofanameToolazytothinkofaname

362 bronze badges




362 bronze badges







  • 11




    $begingroup$
    "a^2 is a factor b^2 and b" You are right $a^2$ divides $b^2$, but why would it divide $b$?
    $endgroup$
    – Wojowu
    Jul 7 at 16:06






  • 7




    $begingroup$
    First error I spotted is that $a^2$ need not be a factor of $b$ just because it is a factor of $b^2$. Indeed, if $a=2$ and $b=2$, then $a^2=4$ is a factor of $b^2=4$ but $a^2=4$ is not a factor of $b=2$.
    $endgroup$
    – Dave
    Jul 7 at 16:06






  • 3




    $begingroup$
    Also, it is clear that integers are rational because given any integer $a$ we can write $a=fraca1$.
    $endgroup$
    – Dave
    Jul 7 at 16:08






  • 4




    $begingroup$
    If b and c are coprime, and a=b/c, then what does c have to be?
    $endgroup$
    – pokep
    Jul 7 at 17:33






  • 3




    $begingroup$
    @Toolazytothinkofaname It is rather simple to spot the error. Substitute the variables by actual integers and find where the proof is wrong.
    $endgroup$
    – miracle173
    Jul 8 at 14:18












  • 11




    $begingroup$
    "a^2 is a factor b^2 and b" You are right $a^2$ divides $b^2$, but why would it divide $b$?
    $endgroup$
    – Wojowu
    Jul 7 at 16:06






  • 7




    $begingroup$
    First error I spotted is that $a^2$ need not be a factor of $b$ just because it is a factor of $b^2$. Indeed, if $a=2$ and $b=2$, then $a^2=4$ is a factor of $b^2=4$ but $a^2=4$ is not a factor of $b=2$.
    $endgroup$
    – Dave
    Jul 7 at 16:06






  • 3




    $begingroup$
    Also, it is clear that integers are rational because given any integer $a$ we can write $a=fraca1$.
    $endgroup$
    – Dave
    Jul 7 at 16:08






  • 4




    $begingroup$
    If b and c are coprime, and a=b/c, then what does c have to be?
    $endgroup$
    – pokep
    Jul 7 at 17:33






  • 3




    $begingroup$
    @Toolazytothinkofaname It is rather simple to spot the error. Substitute the variables by actual integers and find where the proof is wrong.
    $endgroup$
    – miracle173
    Jul 8 at 14:18







11




11




$begingroup$
"a^2 is a factor b^2 and b" You are right $a^2$ divides $b^2$, but why would it divide $b$?
$endgroup$
– Wojowu
Jul 7 at 16:06




$begingroup$
"a^2 is a factor b^2 and b" You are right $a^2$ divides $b^2$, but why would it divide $b$?
$endgroup$
– Wojowu
Jul 7 at 16:06




7




7




$begingroup$
First error I spotted is that $a^2$ need not be a factor of $b$ just because it is a factor of $b^2$. Indeed, if $a=2$ and $b=2$, then $a^2=4$ is a factor of $b^2=4$ but $a^2=4$ is not a factor of $b=2$.
$endgroup$
– Dave
Jul 7 at 16:06




$begingroup$
First error I spotted is that $a^2$ need not be a factor of $b$ just because it is a factor of $b^2$. Indeed, if $a=2$ and $b=2$, then $a^2=4$ is a factor of $b^2=4$ but $a^2=4$ is not a factor of $b=2$.
$endgroup$
– Dave
Jul 7 at 16:06




3




3




$begingroup$
Also, it is clear that integers are rational because given any integer $a$ we can write $a=fraca1$.
$endgroup$
– Dave
Jul 7 at 16:08




$begingroup$
Also, it is clear that integers are rational because given any integer $a$ we can write $a=fraca1$.
$endgroup$
– Dave
Jul 7 at 16:08




4




4




$begingroup$
If b and c are coprime, and a=b/c, then what does c have to be?
$endgroup$
– pokep
Jul 7 at 17:33




$begingroup$
If b and c are coprime, and a=b/c, then what does c have to be?
$endgroup$
– pokep
Jul 7 at 17:33




3




3




$begingroup$
@Toolazytothinkofaname It is rather simple to spot the error. Substitute the variables by actual integers and find where the proof is wrong.
$endgroup$
– miracle173
Jul 8 at 14:18




$begingroup$
@Toolazytothinkofaname It is rather simple to spot the error. Substitute the variables by actual integers and find where the proof is wrong.
$endgroup$
– miracle173
Jul 8 at 14:18










3 Answers
3






active

oldest

votes


















41












$begingroup$

The problem in the proof is that $a^2|b^2nRightarrow a^2|b$. For instance, take $a=2$ and $b=6$. Clearly, $4|36$ but $4nmid 6$.






share|cite|improve this answer









$endgroup$




















    6












    $begingroup$

    I think you are confusing that if $p$ is prime and $p$ divides $b^k$ then $p|b$. That is true if $p$ is prime.



    Actually it's also true for a composite $a|b^k$ then $a|b$ if $a$ has no square factors. But if $a$ as any prime factors to a power greater than $1$ it need not be true.



    And in fact its obviously not true as $a^2$ divides $a^2$ but $a^2$ doesn't divide $a$ (unless $a = 1$).



    Read on....



    It most certainly is not true if $a|b^k$ that $a|b$ It means that the prime factors of $a$ are prime factors of $b$. And it means that the powers of those prime factors of $a$ are at most equal to $k$ times the powers of the same prime factors of $b$ but because $k$ is larger than .....



    Oh let me put it this way.



    Suppose $a = prod p_i^m_i$ be the prime factorization of $a$. Suppose $a|b^k$. Then that means that $p_i$ are prime factors of $b$ and that $b = dprod p_i^j_i$. And it means that $b^k = d^k prod p_i^k*j_i$.



    And as $a|b^k$ that means each $m_i le k*j_i$. But that does not mean $m_i le j_i$ which would mean $a|b$.



    You statement $a|b^k$ means $a|b$ if $a$ has square free and all the prime factor powers were $1$ but not other wise.



    Simple example if $a = 12 = 2^2*3$ and $b= 90 = 2*3^2*5$. Now $a|b^2 = 8100 = 2^2*3^4*5^2$.



    This means the prime factors of $a$ ($2,3$) are also prime factors of $b$. And it means that the powers of the prime factors of $a$ ($2mapsto 2; 3mapsto 1$) are less or equal to $2$ times the powers in $b$ ($2mapsto 1$ and $2 le 2*1$ and $3mapsto 2$ and $1 le 2*2$) but it doesnt mean the are less than or equal to the powers of $b$. (In $a; 2mapsto 2$ but in $b; 2mapsto 1$ and $2 not le 1$).



    So $12 not mid 90$.



    It's certainly can't be the case that $a|b implies a^2| b^2 implies a^2|b$! That would mean every time you have $a|b$ you can just keep squaring and reducing to get $a^m|b$ for any power of $m$.



    That would mean if $3|6$ then $3^2|6$ and $3^4|6$ and $3^2048|6$ and so on.



    Or in this case as $a = b$ (and $c=1$.... because $a$ is an integer) you would have $a|a$ so $a^2|a$? And $a^4|a$. That's .... simply not true.






    share|cite|improve this answer











    $endgroup$












    • $begingroup$
      No, $,pmid b^k,Rightarrow, pmid b,$ is true $iff p,$ is squarefree. Follow the link for a handful of characterizations of squarefree integers.
      $endgroup$
      – Bill Dubuque
      Jul 7 at 17:44











    • $begingroup$
      Why do you say "no"? That is exactly what I said.
      $endgroup$
      – fleablood
      Jul 7 at 17:52










    • $begingroup$
      Because the first paragraph was incorrect. Now it is correct after your edit.
      $endgroup$
      – Bill Dubuque
      Jul 7 at 17:54







    • 1




      $begingroup$
      Okay. I used "only" colloquially. My bad. I'm pretty sure the OP was confusing the FTA with Euclid's lemma. So I said that only works if $p$ is prime. Colloquially that doesn't mean $p$ being prime is required and it is false otherwise. It means you can only cite that lemma if $p$ is prime. There are many other ways $a|b^k$ and $a|b$ can both be true but citing Euclid's lemma is usually reserved for $a$ prime. But in math I shouldn't have used the loaded word "only".
      $endgroup$
      – fleablood
      Jul 7 at 18:56


















    1












    $begingroup$

    Basic facts missing $ac=b$ is a lot easier to use. $a^2$ does not need to divide $b$. A fraction sharing no common factor other than 1, between the number on top ( numerator), and the number on the bottom ( denominator), is said to be in lowest terms .



    Anyways starting from $a=bover c$ we get $ac=b$ showing c divides b, sharing no factor other than 1, and therefore, $c=1$, implying $a=b$ so $a=aover 1$ it Also can be used to show :$a=-aover -1$






    share|cite|improve this answer









    $endgroup$















      Your Answer








      StackExchange.ready(function()
      var channelOptions =
      tags: "".split(" "),
      id: "69"
      ;
      initTagRenderer("".split(" "), "".split(" "), channelOptions);

      StackExchange.using("externalEditor", function()
      // Have to fire editor after snippets, if snippets enabled
      if (StackExchange.settings.snippets.snippetsEnabled)
      StackExchange.using("snippets", function()
      createEditor();
      );

      else
      createEditor();

      );

      function createEditor()
      StackExchange.prepareEditor(
      heartbeatType: 'answer',
      autoActivateHeartbeat: false,
      convertImagesToLinks: true,
      noModals: true,
      showLowRepImageUploadWarning: true,
      reputationToPostImages: 10,
      bindNavPrevention: true,
      postfix: "",
      imageUploader:
      brandingHtml: "Powered by u003ca class="icon-imgur-white" href="https://imgur.com/"u003eu003c/au003e",
      contentPolicyHtml: "User contributions licensed under u003ca href="https://creativecommons.org/licenses/by-sa/3.0/"u003ecc by-sa 3.0 with attribution requiredu003c/au003e u003ca href="https://stackoverflow.com/legal/content-policy"u003e(content policy)u003c/au003e",
      allowUrls: true
      ,
      noCode: true, onDemand: true,
      discardSelector: ".discard-answer"
      ,immediatelyShowMarkdownHelp:true
      );



      );













      draft saved

      draft discarded


















      StackExchange.ready(
      function ()
      StackExchange.openid.initPostLogin('.new-post-login', 'https%3a%2f%2fmath.stackexchange.com%2fquestions%2f3285940%2fwhat-is-the-problem-here-all-integers-are-irrational-proof-i-think-so%23new-answer', 'question_page');

      );

      Post as a guest















      Required, but never shown

























      3 Answers
      3






      active

      oldest

      votes








      3 Answers
      3






      active

      oldest

      votes









      active

      oldest

      votes






      active

      oldest

      votes









      41












      $begingroup$

      The problem in the proof is that $a^2|b^2nRightarrow a^2|b$. For instance, take $a=2$ and $b=6$. Clearly, $4|36$ but $4nmid 6$.






      share|cite|improve this answer









      $endgroup$

















        41












        $begingroup$

        The problem in the proof is that $a^2|b^2nRightarrow a^2|b$. For instance, take $a=2$ and $b=6$. Clearly, $4|36$ but $4nmid 6$.






        share|cite|improve this answer









        $endgroup$















          41












          41








          41





          $begingroup$

          The problem in the proof is that $a^2|b^2nRightarrow a^2|b$. For instance, take $a=2$ and $b=6$. Clearly, $4|36$ but $4nmid 6$.






          share|cite|improve this answer









          $endgroup$



          The problem in the proof is that $a^2|b^2nRightarrow a^2|b$. For instance, take $a=2$ and $b=6$. Clearly, $4|36$ but $4nmid 6$.







          share|cite|improve this answer












          share|cite|improve this answer



          share|cite|improve this answer










          answered Jul 7 at 16:12









          AnandAnand

          5731 silver badge11 bronze badges




          5731 silver badge11 bronze badges























              6












              $begingroup$

              I think you are confusing that if $p$ is prime and $p$ divides $b^k$ then $p|b$. That is true if $p$ is prime.



              Actually it's also true for a composite $a|b^k$ then $a|b$ if $a$ has no square factors. But if $a$ as any prime factors to a power greater than $1$ it need not be true.



              And in fact its obviously not true as $a^2$ divides $a^2$ but $a^2$ doesn't divide $a$ (unless $a = 1$).



              Read on....



              It most certainly is not true if $a|b^k$ that $a|b$ It means that the prime factors of $a$ are prime factors of $b$. And it means that the powers of those prime factors of $a$ are at most equal to $k$ times the powers of the same prime factors of $b$ but because $k$ is larger than .....



              Oh let me put it this way.



              Suppose $a = prod p_i^m_i$ be the prime factorization of $a$. Suppose $a|b^k$. Then that means that $p_i$ are prime factors of $b$ and that $b = dprod p_i^j_i$. And it means that $b^k = d^k prod p_i^k*j_i$.



              And as $a|b^k$ that means each $m_i le k*j_i$. But that does not mean $m_i le j_i$ which would mean $a|b$.



              You statement $a|b^k$ means $a|b$ if $a$ has square free and all the prime factor powers were $1$ but not other wise.



              Simple example if $a = 12 = 2^2*3$ and $b= 90 = 2*3^2*5$. Now $a|b^2 = 8100 = 2^2*3^4*5^2$.



              This means the prime factors of $a$ ($2,3$) are also prime factors of $b$. And it means that the powers of the prime factors of $a$ ($2mapsto 2; 3mapsto 1$) are less or equal to $2$ times the powers in $b$ ($2mapsto 1$ and $2 le 2*1$ and $3mapsto 2$ and $1 le 2*2$) but it doesnt mean the are less than or equal to the powers of $b$. (In $a; 2mapsto 2$ but in $b; 2mapsto 1$ and $2 not le 1$).



              So $12 not mid 90$.



              It's certainly can't be the case that $a|b implies a^2| b^2 implies a^2|b$! That would mean every time you have $a|b$ you can just keep squaring and reducing to get $a^m|b$ for any power of $m$.



              That would mean if $3|6$ then $3^2|6$ and $3^4|6$ and $3^2048|6$ and so on.



              Or in this case as $a = b$ (and $c=1$.... because $a$ is an integer) you would have $a|a$ so $a^2|a$? And $a^4|a$. That's .... simply not true.






              share|cite|improve this answer











              $endgroup$












              • $begingroup$
                No, $,pmid b^k,Rightarrow, pmid b,$ is true $iff p,$ is squarefree. Follow the link for a handful of characterizations of squarefree integers.
                $endgroup$
                – Bill Dubuque
                Jul 7 at 17:44











              • $begingroup$
                Why do you say "no"? That is exactly what I said.
                $endgroup$
                – fleablood
                Jul 7 at 17:52










              • $begingroup$
                Because the first paragraph was incorrect. Now it is correct after your edit.
                $endgroup$
                – Bill Dubuque
                Jul 7 at 17:54







              • 1




                $begingroup$
                Okay. I used "only" colloquially. My bad. I'm pretty sure the OP was confusing the FTA with Euclid's lemma. So I said that only works if $p$ is prime. Colloquially that doesn't mean $p$ being prime is required and it is false otherwise. It means you can only cite that lemma if $p$ is prime. There are many other ways $a|b^k$ and $a|b$ can both be true but citing Euclid's lemma is usually reserved for $a$ prime. But in math I shouldn't have used the loaded word "only".
                $endgroup$
                – fleablood
                Jul 7 at 18:56















              6












              $begingroup$

              I think you are confusing that if $p$ is prime and $p$ divides $b^k$ then $p|b$. That is true if $p$ is prime.



              Actually it's also true for a composite $a|b^k$ then $a|b$ if $a$ has no square factors. But if $a$ as any prime factors to a power greater than $1$ it need not be true.



              And in fact its obviously not true as $a^2$ divides $a^2$ but $a^2$ doesn't divide $a$ (unless $a = 1$).



              Read on....



              It most certainly is not true if $a|b^k$ that $a|b$ It means that the prime factors of $a$ are prime factors of $b$. And it means that the powers of those prime factors of $a$ are at most equal to $k$ times the powers of the same prime factors of $b$ but because $k$ is larger than .....



              Oh let me put it this way.



              Suppose $a = prod p_i^m_i$ be the prime factorization of $a$. Suppose $a|b^k$. Then that means that $p_i$ are prime factors of $b$ and that $b = dprod p_i^j_i$. And it means that $b^k = d^k prod p_i^k*j_i$.



              And as $a|b^k$ that means each $m_i le k*j_i$. But that does not mean $m_i le j_i$ which would mean $a|b$.



              You statement $a|b^k$ means $a|b$ if $a$ has square free and all the prime factor powers were $1$ but not other wise.



              Simple example if $a = 12 = 2^2*3$ and $b= 90 = 2*3^2*5$. Now $a|b^2 = 8100 = 2^2*3^4*5^2$.



              This means the prime factors of $a$ ($2,3$) are also prime factors of $b$. And it means that the powers of the prime factors of $a$ ($2mapsto 2; 3mapsto 1$) are less or equal to $2$ times the powers in $b$ ($2mapsto 1$ and $2 le 2*1$ and $3mapsto 2$ and $1 le 2*2$) but it doesnt mean the are less than or equal to the powers of $b$. (In $a; 2mapsto 2$ but in $b; 2mapsto 1$ and $2 not le 1$).



              So $12 not mid 90$.



              It's certainly can't be the case that $a|b implies a^2| b^2 implies a^2|b$! That would mean every time you have $a|b$ you can just keep squaring and reducing to get $a^m|b$ for any power of $m$.



              That would mean if $3|6$ then $3^2|6$ and $3^4|6$ and $3^2048|6$ and so on.



              Or in this case as $a = b$ (and $c=1$.... because $a$ is an integer) you would have $a|a$ so $a^2|a$? And $a^4|a$. That's .... simply not true.






              share|cite|improve this answer











              $endgroup$












              • $begingroup$
                No, $,pmid b^k,Rightarrow, pmid b,$ is true $iff p,$ is squarefree. Follow the link for a handful of characterizations of squarefree integers.
                $endgroup$
                – Bill Dubuque
                Jul 7 at 17:44











              • $begingroup$
                Why do you say "no"? That is exactly what I said.
                $endgroup$
                – fleablood
                Jul 7 at 17:52










              • $begingroup$
                Because the first paragraph was incorrect. Now it is correct after your edit.
                $endgroup$
                – Bill Dubuque
                Jul 7 at 17:54







              • 1




                $begingroup$
                Okay. I used "only" colloquially. My bad. I'm pretty sure the OP was confusing the FTA with Euclid's lemma. So I said that only works if $p$ is prime. Colloquially that doesn't mean $p$ being prime is required and it is false otherwise. It means you can only cite that lemma if $p$ is prime. There are many other ways $a|b^k$ and $a|b$ can both be true but citing Euclid's lemma is usually reserved for $a$ prime. But in math I shouldn't have used the loaded word "only".
                $endgroup$
                – fleablood
                Jul 7 at 18:56













              6












              6








              6





              $begingroup$

              I think you are confusing that if $p$ is prime and $p$ divides $b^k$ then $p|b$. That is true if $p$ is prime.



              Actually it's also true for a composite $a|b^k$ then $a|b$ if $a$ has no square factors. But if $a$ as any prime factors to a power greater than $1$ it need not be true.



              And in fact its obviously not true as $a^2$ divides $a^2$ but $a^2$ doesn't divide $a$ (unless $a = 1$).



              Read on....



              It most certainly is not true if $a|b^k$ that $a|b$ It means that the prime factors of $a$ are prime factors of $b$. And it means that the powers of those prime factors of $a$ are at most equal to $k$ times the powers of the same prime factors of $b$ but because $k$ is larger than .....



              Oh let me put it this way.



              Suppose $a = prod p_i^m_i$ be the prime factorization of $a$. Suppose $a|b^k$. Then that means that $p_i$ are prime factors of $b$ and that $b = dprod p_i^j_i$. And it means that $b^k = d^k prod p_i^k*j_i$.



              And as $a|b^k$ that means each $m_i le k*j_i$. But that does not mean $m_i le j_i$ which would mean $a|b$.



              You statement $a|b^k$ means $a|b$ if $a$ has square free and all the prime factor powers were $1$ but not other wise.



              Simple example if $a = 12 = 2^2*3$ and $b= 90 = 2*3^2*5$. Now $a|b^2 = 8100 = 2^2*3^4*5^2$.



              This means the prime factors of $a$ ($2,3$) are also prime factors of $b$. And it means that the powers of the prime factors of $a$ ($2mapsto 2; 3mapsto 1$) are less or equal to $2$ times the powers in $b$ ($2mapsto 1$ and $2 le 2*1$ and $3mapsto 2$ and $1 le 2*2$) but it doesnt mean the are less than or equal to the powers of $b$. (In $a; 2mapsto 2$ but in $b; 2mapsto 1$ and $2 not le 1$).



              So $12 not mid 90$.



              It's certainly can't be the case that $a|b implies a^2| b^2 implies a^2|b$! That would mean every time you have $a|b$ you can just keep squaring and reducing to get $a^m|b$ for any power of $m$.



              That would mean if $3|6$ then $3^2|6$ and $3^4|6$ and $3^2048|6$ and so on.



              Or in this case as $a = b$ (and $c=1$.... because $a$ is an integer) you would have $a|a$ so $a^2|a$? And $a^4|a$. That's .... simply not true.






              share|cite|improve this answer











              $endgroup$



              I think you are confusing that if $p$ is prime and $p$ divides $b^k$ then $p|b$. That is true if $p$ is prime.



              Actually it's also true for a composite $a|b^k$ then $a|b$ if $a$ has no square factors. But if $a$ as any prime factors to a power greater than $1$ it need not be true.



              And in fact its obviously not true as $a^2$ divides $a^2$ but $a^2$ doesn't divide $a$ (unless $a = 1$).



              Read on....



              It most certainly is not true if $a|b^k$ that $a|b$ It means that the prime factors of $a$ are prime factors of $b$. And it means that the powers of those prime factors of $a$ are at most equal to $k$ times the powers of the same prime factors of $b$ but because $k$ is larger than .....



              Oh let me put it this way.



              Suppose $a = prod p_i^m_i$ be the prime factorization of $a$. Suppose $a|b^k$. Then that means that $p_i$ are prime factors of $b$ and that $b = dprod p_i^j_i$. And it means that $b^k = d^k prod p_i^k*j_i$.



              And as $a|b^k$ that means each $m_i le k*j_i$. But that does not mean $m_i le j_i$ which would mean $a|b$.



              You statement $a|b^k$ means $a|b$ if $a$ has square free and all the prime factor powers were $1$ but not other wise.



              Simple example if $a = 12 = 2^2*3$ and $b= 90 = 2*3^2*5$. Now $a|b^2 = 8100 = 2^2*3^4*5^2$.



              This means the prime factors of $a$ ($2,3$) are also prime factors of $b$. And it means that the powers of the prime factors of $a$ ($2mapsto 2; 3mapsto 1$) are less or equal to $2$ times the powers in $b$ ($2mapsto 1$ and $2 le 2*1$ and $3mapsto 2$ and $1 le 2*2$) but it doesnt mean the are less than or equal to the powers of $b$. (In $a; 2mapsto 2$ but in $b; 2mapsto 1$ and $2 not le 1$).



              So $12 not mid 90$.



              It's certainly can't be the case that $a|b implies a^2| b^2 implies a^2|b$! That would mean every time you have $a|b$ you can just keep squaring and reducing to get $a^m|b$ for any power of $m$.



              That would mean if $3|6$ then $3^2|6$ and $3^4|6$ and $3^2048|6$ and so on.



              Or in this case as $a = b$ (and $c=1$.... because $a$ is an integer) you would have $a|a$ so $a^2|a$? And $a^4|a$. That's .... simply not true.







              share|cite|improve this answer














              share|cite|improve this answer



              share|cite|improve this answer








              edited Jul 7 at 17:52

























              answered Jul 7 at 17:23









              fleabloodfleablood

              76k2 gold badges28 silver badges95 bronze badges




              76k2 gold badges28 silver badges95 bronze badges











              • $begingroup$
                No, $,pmid b^k,Rightarrow, pmid b,$ is true $iff p,$ is squarefree. Follow the link for a handful of characterizations of squarefree integers.
                $endgroup$
                – Bill Dubuque
                Jul 7 at 17:44











              • $begingroup$
                Why do you say "no"? That is exactly what I said.
                $endgroup$
                – fleablood
                Jul 7 at 17:52










              • $begingroup$
                Because the first paragraph was incorrect. Now it is correct after your edit.
                $endgroup$
                – Bill Dubuque
                Jul 7 at 17:54







              • 1




                $begingroup$
                Okay. I used "only" colloquially. My bad. I'm pretty sure the OP was confusing the FTA with Euclid's lemma. So I said that only works if $p$ is prime. Colloquially that doesn't mean $p$ being prime is required and it is false otherwise. It means you can only cite that lemma if $p$ is prime. There are many other ways $a|b^k$ and $a|b$ can both be true but citing Euclid's lemma is usually reserved for $a$ prime. But in math I shouldn't have used the loaded word "only".
                $endgroup$
                – fleablood
                Jul 7 at 18:56
















              • $begingroup$
                No, $,pmid b^k,Rightarrow, pmid b,$ is true $iff p,$ is squarefree. Follow the link for a handful of characterizations of squarefree integers.
                $endgroup$
                – Bill Dubuque
                Jul 7 at 17:44











              • $begingroup$
                Why do you say "no"? That is exactly what I said.
                $endgroup$
                – fleablood
                Jul 7 at 17:52










              • $begingroup$
                Because the first paragraph was incorrect. Now it is correct after your edit.
                $endgroup$
                – Bill Dubuque
                Jul 7 at 17:54







              • 1




                $begingroup$
                Okay. I used "only" colloquially. My bad. I'm pretty sure the OP was confusing the FTA with Euclid's lemma. So I said that only works if $p$ is prime. Colloquially that doesn't mean $p$ being prime is required and it is false otherwise. It means you can only cite that lemma if $p$ is prime. There are many other ways $a|b^k$ and $a|b$ can both be true but citing Euclid's lemma is usually reserved for $a$ prime. But in math I shouldn't have used the loaded word "only".
                $endgroup$
                – fleablood
                Jul 7 at 18:56















              $begingroup$
              No, $,pmid b^k,Rightarrow, pmid b,$ is true $iff p,$ is squarefree. Follow the link for a handful of characterizations of squarefree integers.
              $endgroup$
              – Bill Dubuque
              Jul 7 at 17:44





              $begingroup$
              No, $,pmid b^k,Rightarrow, pmid b,$ is true $iff p,$ is squarefree. Follow the link for a handful of characterizations of squarefree integers.
              $endgroup$
              – Bill Dubuque
              Jul 7 at 17:44













              $begingroup$
              Why do you say "no"? That is exactly what I said.
              $endgroup$
              – fleablood
              Jul 7 at 17:52




              $begingroup$
              Why do you say "no"? That is exactly what I said.
              $endgroup$
              – fleablood
              Jul 7 at 17:52












              $begingroup$
              Because the first paragraph was incorrect. Now it is correct after your edit.
              $endgroup$
              – Bill Dubuque
              Jul 7 at 17:54





              $begingroup$
              Because the first paragraph was incorrect. Now it is correct after your edit.
              $endgroup$
              – Bill Dubuque
              Jul 7 at 17:54





              1




              1




              $begingroup$
              Okay. I used "only" colloquially. My bad. I'm pretty sure the OP was confusing the FTA with Euclid's lemma. So I said that only works if $p$ is prime. Colloquially that doesn't mean $p$ being prime is required and it is false otherwise. It means you can only cite that lemma if $p$ is prime. There are many other ways $a|b^k$ and $a|b$ can both be true but citing Euclid's lemma is usually reserved for $a$ prime. But in math I shouldn't have used the loaded word "only".
              $endgroup$
              – fleablood
              Jul 7 at 18:56




              $begingroup$
              Okay. I used "only" colloquially. My bad. I'm pretty sure the OP was confusing the FTA with Euclid's lemma. So I said that only works if $p$ is prime. Colloquially that doesn't mean $p$ being prime is required and it is false otherwise. It means you can only cite that lemma if $p$ is prime. There are many other ways $a|b^k$ and $a|b$ can both be true but citing Euclid's lemma is usually reserved for $a$ prime. But in math I shouldn't have used the loaded word "only".
              $endgroup$
              – fleablood
              Jul 7 at 18:56











              1












              $begingroup$

              Basic facts missing $ac=b$ is a lot easier to use. $a^2$ does not need to divide $b$. A fraction sharing no common factor other than 1, between the number on top ( numerator), and the number on the bottom ( denominator), is said to be in lowest terms .



              Anyways starting from $a=bover c$ we get $ac=b$ showing c divides b, sharing no factor other than 1, and therefore, $c=1$, implying $a=b$ so $a=aover 1$ it Also can be used to show :$a=-aover -1$






              share|cite|improve this answer









              $endgroup$

















                1












                $begingroup$

                Basic facts missing $ac=b$ is a lot easier to use. $a^2$ does not need to divide $b$. A fraction sharing no common factor other than 1, between the number on top ( numerator), and the number on the bottom ( denominator), is said to be in lowest terms .



                Anyways starting from $a=bover c$ we get $ac=b$ showing c divides b, sharing no factor other than 1, and therefore, $c=1$, implying $a=b$ so $a=aover 1$ it Also can be used to show :$a=-aover -1$






                share|cite|improve this answer









                $endgroup$















                  1












                  1








                  1





                  $begingroup$

                  Basic facts missing $ac=b$ is a lot easier to use. $a^2$ does not need to divide $b$. A fraction sharing no common factor other than 1, between the number on top ( numerator), and the number on the bottom ( denominator), is said to be in lowest terms .



                  Anyways starting from $a=bover c$ we get $ac=b$ showing c divides b, sharing no factor other than 1, and therefore, $c=1$, implying $a=b$ so $a=aover 1$ it Also can be used to show :$a=-aover -1$






                  share|cite|improve this answer









                  $endgroup$



                  Basic facts missing $ac=b$ is a lot easier to use. $a^2$ does not need to divide $b$. A fraction sharing no common factor other than 1, between the number on top ( numerator), and the number on the bottom ( denominator), is said to be in lowest terms .



                  Anyways starting from $a=bover c$ we get $ac=b$ showing c divides b, sharing no factor other than 1, and therefore, $c=1$, implying $a=b$ so $a=aover 1$ it Also can be used to show :$a=-aover -1$







                  share|cite|improve this answer












                  share|cite|improve this answer



                  share|cite|improve this answer










                  answered Jul 8 at 0:41









                  Roddy MacPheeRoddy MacPhee

                  282 gold badges2 silver badges25 bronze badges




                  282 gold badges2 silver badges25 bronze badges



























                      draft saved

                      draft discarded
















































                      Thanks for contributing an answer to Mathematics Stack Exchange!


                      • Please be sure to answer the question. Provide details and share your research!

                      But avoid


                      • Asking for help, clarification, or responding to other answers.

                      • Making statements based on opinion; back them up with references or personal experience.

                      Use MathJax to format equations. MathJax reference.


                      To learn more, see our tips on writing great answers.




                      draft saved


                      draft discarded














                      StackExchange.ready(
                      function ()
                      StackExchange.openid.initPostLogin('.new-post-login', 'https%3a%2f%2fmath.stackexchange.com%2fquestions%2f3285940%2fwhat-is-the-problem-here-all-integers-are-irrational-proof-i-think-so%23new-answer', 'question_page');

                      );

                      Post as a guest















                      Required, but never shown





















































                      Required, but never shown














                      Required, but never shown












                      Required, but never shown







                      Required, but never shown

































                      Required, but never shown














                      Required, but never shown












                      Required, but never shown







                      Required, but never shown







                      Popular posts from this blog

                      Get product attribute by attribute group code in magento 2get product attribute by product attribute group in magento 2Magento 2 Log Bundle Product Data in List Page?How to get all product attribute of a attribute group of Default attribute set?Magento 2.1 Create a filter in the product grid by new attributeMagento 2 : Get Product Attribute values By GroupMagento 2 How to get all existing values for one attributeMagento 2 get custom attribute of a single product inside a pluginMagento 2.3 How to get all the Multi Source Inventory (MSI) locations collection in custom module?Magento2: how to develop rest API to get new productsGet product attribute by attribute group code ( [attribute_group_code] ) in magento 2

                      Category:9 (number) SubcategoriesMedia in category "9 (number)"Navigation menuUpload mediaGND ID: 4485639-8Library of Congress authority ID: sh85091979ReasonatorScholiaStatistics

                      Magento 2.3: How do i solve this, Not registered handle, on custom form?How can i rewrite TierPrice Block in Magento2magento 2 captcha not rendering if I override layout xmlmain.CRITICAL: Plugin class doesn't existMagento 2 : Problem while adding custom button order view page?Magento 2.2.5: Overriding Admin Controller sales/orderMagento 2.2.5: Add, Update and Delete existing products Custom OptionsMagento 2.3 : File Upload issue in UI Component FormMagento2 Not registered handleHow to configured Form Builder Js in my custom magento 2.3.0 module?Magento 2.3. How to create image upload field in an admin form